Proof: P is an Orthogonal Projection with P^2=P

Click For Summary
SUMMARY

The discussion confirms that for a linear operator P on a vector space V, if P satisfies the condition P^2 = P, it is necessary to also demonstrate that ||Pv|| ≤ ||v|| to establish that P is an orthogonal projection. A counterexample is provided using an oblique projection matrix in ℝ², specifically P = [[0, 0], [c, 1]], which shows that the first condition alone does not guarantee the orthogonality of the projection. The second condition is essential to ensure that the projection does not increase the magnitude of the vector.

PREREQUISITES
  • Understanding of linear operators and vector spaces
  • Familiarity with the concept of orthogonal projections
  • Knowledge of matrix operations and properties
  • Basic understanding of norms and vector magnitudes
NEXT STEPS
  • Study the properties of orthogonal projections in linear algebra
  • Learn about oblique projections and their characteristics
  • Explore the implications of the projection theorem in vector spaces
  • Investigate the relationship between linear transformations and their matrix representations
USEFUL FOR

Students and professionals in mathematics, particularly those studying linear algebra, as well as educators seeking to clarify the distinctions between orthogonal and oblique projections.

evilpostingmong
Messages
338
Reaction score
0

Homework Statement


Let P\inL(V). If P^2=P, and llPvll<=llvll, prove that P is an orthogonal projection.

Homework Equations


The Attempt at a Solution


I think that regarding llPvll<=llvll is redundant. For example, consider P^2=P
and let v be a vector in V. Doesn't P^2=P kind of give it away by itself?
I mean v=a1v1+...+amvm+...+anvn. Consider the subspace that P projects to whose
dimension is less than V's. So for P^2 to =P, and P^2v=/=0,
PPv=a1v1+...+amvm=Pv=a1v1+...+amvm. Notice how the scalars from 1 to m
do not change to make P^2=P possible. Isn't it obvious enough from P^2=P
that the length of the vector a1v1+..+amvm is < a1v1+...+amvm+...+anvn?
I know that that's not were trying to prove, but why is llPvll<=llvll important
when we know P^2=P?
 
Last edited:
Physics news on Phys.org
The condition is not redundant. The condition that P^2=P is precisely the definition of a projection, but you most definitely need the second piece, that ||Pv||\leq||v|| to show that it is orthogonal. As a counterexample, consider the oblique (as opposed to orthogonal) projection in \mathbb{R}^2

P=\[ \left( \begin{array}{ccc}<br /> 0 &amp; 0 \\<br /> c &amp; 1 \end{array} \right)\]

You can quickly check that it is indeed a projection by computing P^2=P but if we look at its action on a vector we find:

Pv=\[ \left( \begin{array}{ccc}<br /> 0 \\<br /> cv_1+v_2 \end{array} \right)\]<br />

So for c>1, we would have ||Pv||&gt;||v||. This goes to show that an projection need not always reduce the magnitude of a vector upon which it acts and hence the second condition is not derivable from the first. In fact, we have to hope that the added condition is precisely what you need to show that the projection is orthogonal, or else that's going to be one tricky homework problem :wink:
 
cipher42 said:
The condition is not redundant. The condition that P^2=P is precisely the definition of a projection, but you most definitely need the second piece, that ||Pv||\leq||v|| to show that it is orthogonal. As a counterexample, consider the oblique (as opposed to orthogonal) projection in \mathbb{R}^2

P=\[ \left( \begin{array}{ccc}<br /> 0 &amp; 0 \\<br /> c &amp; 1 \end{array} \right)\]

You can quickly check that it is indeed a projection by computing P^2=P but if we look at its action on a vector we find:

Pv=\[ \left( \begin{array}{ccc}<br /> 0 \\<br /> cv_1+v_2 \end{array} \right)\]<br />

So for c>1, we would have ||Pv||&gt;||v||. This goes to show that an projection need not always reduce the magnitude of a vector upon which it acts and hence the second condition is not derivable from the first. In fact, we have to hope that the added condition is precisely what you need to show that the projection is orthogonal, or else that's going to be one tricky homework problem :wink:

Thank you! There had to be some motive for the author to put llPvll<=llvll, I just
couldn't find it.
 
Question: A clock's minute hand has length 4 and its hour hand has length 3. What is the distance between the tips at the moment when it is increasing most rapidly?(Putnam Exam Question) Answer: Making assumption that both the hands moves at constant angular velocities, the answer is ## \sqrt{7} .## But don't you think this assumption is somewhat doubtful and wrong?

Similar threads

  • · Replies 1 ·
Replies
1
Views
1K
Replies
5
Views
2K
Replies
1
Views
1K
  • · Replies 13 ·
Replies
13
Views
6K
  • · Replies 5 ·
Replies
5
Views
2K
Replies
5
Views
2K
Replies
11
Views
14K
Replies
2
Views
1K
  • · Replies 3 ·
Replies
3
Views
4K
Replies
1
Views
1K